LSAT and Law School Admissions Forum

Get expert LSAT preparation and law school admissions advice from PowerScore Test Preparation.

 Administrator
PowerScore Staff
  • PowerScore Staff
  • Posts: 8917
  • Joined: Feb 02, 2011
|
#38148
Complete Question Explanation
(The complete setup for this game can be found here: lsat/viewtopic.php?t=14969)

The correct answer choice is (B)

This question establishes that the officer who evaluates M does not evaluate any other applications. This officer cannot be U (because U evaluates G) or S (because S evaluates at least two applicants). Therefore, the officer who evaluates M must be either R or T:
PT71_D13 LG Explanations_Game #2_#9_diagram 1.png
Given our inference requiring the FL block to be evaluated by either R or S, we can conclude that in this case, both F and L must be evaluated by S:
PT71_D13 LG Explanations_Game #2_#9_diagram 2.png
Our local diagram shows that answer choice (B) must be true, and is therefore correct. Answer choices (A), (C), (D), and (E) could be true, but do not need to be true.
You do not have the required permissions to view the files attached to this post.

Get the most out of your LSAT Prep Plus subscription.

Analyze and track your performance with our Testing and Analytics Package.